Jump to content

Photo

Đề thi chọn đội tuyển Quốc gia tỉnh Vũng Tàu năm 2016-2017

vungtautst

  • Please log in to reply
14 replies to this topic

#1
namcpnh

namcpnh

    Red Devil

  • Hiệp sỹ
  • 1153 posts

Lười gõ, gửi ảnh, anh em tham khảo :D

Nguồn: Phạm Hồng Nhung, c LQĐ VT

 

NbaaffK.jpg


Edited by namcpnh, 27-10-2016 - 20:24.

Cùng chung sức làm chuyên đề hay cho diễn đàn tại :

Dãy số-giới hạn, Đa thức , Hình học , Phương trình hàm , PT-HPT-BPT , Số học.

Wolframalpha đây


#2
pdtienArsFC

pdtienArsFC

    Trung sĩ

  • Thành viên
  • 133 posts

Bài 4: 

2.      $f(x+yf(x))=xf(y)+f(x)$             (1)        

P(x,y) là phép thế của x,y vào (1)

Gọi a là số thỏa mãn $f(a)=0$.

$P(a,y) \Rightarrow af(y)=0 \Rightarrow a=0$.

Vậy $f(x)=0\Leftrightarrow x=0$

$P(-1,-1) \Rightarrow f(-1)=-1$

$P(1,-1) \Rightarrow f(1-f(1))=f(1)-1$

$P(1-f(1),1) \Rightarrow (f(1)-1)^2=0 \Rightarrow f(1)=1$

$P(1,y) \Rightarrow f(y+1)=f(y)+1$

$P(x, y+1) \Rightarrow f(x+yf(x)+f(x))=xf(y)+f(x)+x=f(x+yf(x))+x$    (2)

Mặt khác vì $x+yf(x)$ toàn ánh( cho x=1, y chạy) nên thay $x+yf(x)$ bởi $y$ ở (2), ta có:

$f(y+f(x))=f(y)+x$               (3)

Thay $y=0$ vào (3) ta có: $f(f(x))=x$

Thay $x$ bởi $f(x)$ vào (3) ta có: $f(x+y)=f(x)+f(y)$        (4)

$P(f(x),y)$ kết hợp (4) ta có: $f(xy)=f(x).f(y)$      (5)

Từ (4),(5) ta có nghiệm hàm là $f(x)=x$ và $f(x) \equiv 0$. 


Edited by pdtienArsFC, 05-10-2016 - 11:47.

                           80b68e1e79774daab705a98543684359.0.gif

 


#3
moonkey01

moonkey01

    Hạ sĩ

  • Thành viên
  • 50 posts

Bài 4: 

2.      $f(x+yf(x))=xf(y)+f(x)$             (1)        

P(x,y) là phép thế của x,y vào (1)

Gọi a là số thỏa mãn $f(a)=0$.

$P(a,y) \Rightarrow af(y)=0 \Rightarrow a=0$.

Vậy $f(x)=0\Leftrightarrow x=0$

$P(-1,-1) \Rightarrow f(-1)=-1$

$P(1,-1) \Rightarrow f(1-f(1))=f(1)-1$

$P(1-f(1),1) \Rightarrow (f(1)-1)^2=0 \Rightarrow f(1)=1$

$P(1,y) \Rightarrow f(y+1)=f(y)+1$

$P(x, y+1) \Rightarrow f(x+yf(x)+f(x))=xf(y)+f(x)+x=f(x+yf(x))+x$    (2)

Mặt khác vì $x+yf(x)$ toàn ánh( cho x=1, y chạy) nên thay $x+yf(x)$ bởi $y$ ở (2), ta có:

$f(y+f(x))=f(y)+x$               (3)

Thay $y=0$ vào (3) ta có: $f(f(x))=x$

Thay $x$ bởi $f(x)$ vào (3) ta có: $f(x+y)=f(x)+f(y)$        (4)

$P(f(x),y)$ $x+yf(x)$ kết hợp (4) ta có: $f(xy)=f(x).f(y)$      (5)

Từ (4),(5) ta có nghiệm hàm là $f(x)=x$ và $f(x) \equiv 0$. 

 

Nếu như không tồn tại $a$ để $f(a)=0$ thì sao ? Bạn phải chỉ ra được rằng tồn tại số $a$ như thế trước khi thế vào phương trình ban đầu.


Edited by moonkey01, 04-10-2016 - 18:14.


#4
moonkey01

moonkey01

    Hạ sĩ

  • Thành viên
  • 50 posts

Bài 4a:

 

Vế phải của phương trình ban đầu là một hàm bậc nhất theo $y$ nên $f$ là toàn ánh.

 

Lại thay $x$ bởi $f(x)$ vào phương trình ban đầu, dễ chứng minh được $f$ là đơn ánh, hay $f$ là song ánh.

 

Do đó $\exists! a$ sao cho $f(a)=0$. Thay $y$ bởi $a$ vào phương trình ban đầu, ta có $f(x)=a-2017f(x)$ hay $f(x)=c$ ($c$ là hằng số thực).

 

Thử lại ta thấy không thoả mãn, vậy không tồn tại hàm số thoả mãn yêu cầu đề bài.


Edited by moonkey01, 04-10-2016 - 18:26.


#5
pdtienArsFC

pdtienArsFC

    Trung sĩ

  • Thành viên
  • 133 posts

Nếu như không tồn tại $a$ để $f(a)=0$ thì sao ? Bạn phải chỉ ra được rằng tồn tại số $a$ như thế trước khi thế vào phương trình ban đầu.

Thay $y=0$ ta thấy $f(0)=0$.

Giả sử tồn tại a khác 0 thỏa mãn $f(a)=0$.

.....

Thế đã được chưa em??


                           80b68e1e79774daab705a98543684359.0.gif

 


#6
moonkey01

moonkey01

    Hạ sĩ

  • Thành viên
  • 50 posts

Bài 1a:

 

Với $x,y,z$ là các số thực dương, theo bất đẳng thức AM-GM, ta có:

 

$\sum \frac{1}{(2x+y+z)^2}=\sum \frac{1}{((x+y)+(x+z))^2}\leq \sum \frac{1}{4(x+y)(x+z)}$

 

Do đó ta cần chứng minh rằng $\sum \frac{1}{4(x+y)(x+z)}\leq \frac{3}{16}$ hay $\frac{8}{3}(x+y+z)\leq (x+y)(y+z)(z+x)$

 

Theo một kết quả quen thuộc và AM-GM, $(x+y)(y+z)(z+x)\geq \frac{8}{9}(a+b+c)(ab+bc+ca)\geq \frac{8}{9}(a+b+c)\sqrt[3]{a^2b^2c^2}=\frac{8}{3}(x+y+z)$.

 

Từ đó ta có điều phải chứng minh.



#7
superpower

superpower

    Sĩ quan

  • Thành viên
  • 492 posts

Cho $x,y,z$ không âm và thỏa $x^2+y^2+z^2=1 $

Chứng minh rằng $(x^2y+y^2z+z^2x)(\dfrac{1}{\sqrt{x^2+1}}+\frac{1}{\sqrt{y^2+1}}+\frac{1}{\sqrt{z^2+1}}) \leq \frac{3}{2}$ 

Bài làm

Ta có $(x^2y+y^2z+z^2x )^2 \leq (x^2y^2+y^2z^2+z^2x^2)(x^2+y^2+z^2) =x^2y^2+y^2z^2+z^2x^2 $

Do đó, ta đặt $a=x^2, b=y^2, c=z^2 $ với $a+b+c=1 $

BĐT cần chứng minh sẽ có dạng 

$\sqrt{ab+bc+ca} (\frac{1}{\sqrt{a+1}}+\frac{1}{\sqrt{b+1}}+\frac{1}{\sqrt{c+1}} ) \leq \frac{3}{2} $ 

Bình phương 2 vế và sử dụng bđt AM-GM, ta cần chứng minh 

$(ab+bc+ca)(\frac{1}{1+a} + \frac{1}{1+b} + \frac{1}{1+c} )\leq \frac{3}{4} $

Quy $PQR$, ta cần chứng minh 

$q. \frac{q+2p+3}{1+p+q+r} \leq \frac{3}{4} $

Hay là $r \geq \frac{4q^2+17q-6}{3} $ 

Mặt khác ,ta có bđt Schur bậc 3: $p^3-4pq+9r \geq 0 <=> r \geq \frac{4q-1}{9} $ 

Do đó, ta chỉ cần chứng minh bđt sau là đủ

$\frac{4q-1}{9} \geq \frac{4q^2+17q-6}{9} <=> (3q-1)(4q+17) \leq 0 $ 

Mà này hiển nhiên đúng theo AM-GM thì $ 0 <q \leq \frac{1}{3} $

Do đó, bất đẳng thức được chứng minh

Dấu bằng xảy ra khi $x=y=z $


Edited by superpower, 04-10-2016 - 19:32.


#8
phamngochung9a

phamngochung9a

    Sĩ quan

  • Điều hành viên THPT
  • 480 posts

Cách khác cho Bài 1. 2) 

 

Ta có:

 

$x+y+z=\left ( x^{2}+y^{2}+z^{2} \right )\left ( x+y+z \right )\geq 3\left ( x^{2}y+y^{2}z+z^{2}x \right )$

 

Do đó:

$VT\leq \frac{x+y+z}{3}\left ( \sum \frac{1}{\sqrt{1+x^{2}}} \right )$

 

Bằng phương pháp tiếp tuyến dễ dàng chỉ ra $\frac{1}{\sqrt{1+a^{2}}}\leq -\frac{3}{8}a+\frac{5\sqrt{3}}{8}$    $\forall a\in \left [ 0;1 \right ]$

 

Vậy:

$VT\leq \frac{x+y+z}{3}.\left [-\frac{3}{8}\left ( x+y+z \right )+\frac{15\sqrt{3}}{8} \right ]\\=-\frac{1}{8}\left ( x+y+z \right )^{2}+\frac{5\sqrt{3}}{8}\left ( x+y+z \right )\\=-\frac{1}{8}\left ( x+y+z-4\sqrt{3} \right )\left ( x+y+z-\sqrt{3} \right )+\frac{3}{2}$

 

Vì $x+y+z\leq \sqrt{3}$ nên

 

$VT\leq \frac{3}{2}=VP\rightarrow Q.E.D$

 

Đẳng thức xảy ra $\Leftrightarrow x=y=z=\frac{1}{\sqrt{3}}$


Edited by phamngochung9a, 04-10-2016 - 19:52.


#9
superpower

superpower

    Sĩ quan

  • Thành viên
  • 492 posts

Cho mình hỏi là ở câu dãy 2b, $k$ cố định à. Đề mờ quá không thấy



#10
superpower

superpower

    Sĩ quan

  • Thành viên
  • 492 posts

Bài 4a:

 

Vế phải của phương trình ban đầu là một hàm bậc nhất theo $y$ nên $f$ là toàn ánh.

 

Lại thay $x$ bởi $f(x)$ vào phương trình ban đầu, dễ chứng minh được $f$ là đơn ánh, hay $f$ là song ánh.

 

Do đó $\exists! a$ sao cho $f(a)=0$. Thay $y$ bởi $a$ vào phương trình ban đầu, ta có $f(x)=a-2017f(x)$ hay $f(x)=c$ ($c$ là hằng số thực).

 

Thử lại ta thấy không thoả mãn, vậy không tồn tại hàm số thoả mãn yêu cầu đề bài.

Không làm vậy được nhé bạn

$f(x-2016f(y)) = y-2017f(x) $

Chỉ suy ra đc $f(u) -2017f(v) $ toàn ánh thôi 

Bài này ta làm như sau

Thay $x=2016f(y) => f(0) = y -2017f(2016f(y)) $

Từ đây suy ra $f$ là song ánh 

Do đó tồn tại $a: f(a)=0 $

Thay $y=a => f(x)  = a - 2017f(x) => f(x) =c $

Thử lại thấy không thỏa

Do đó không tồn tại hàm thỏa bài toán



#11
The God of Math

The God of Math

    Lính mới

  • Thành viên
  • 6 posts

Câu 1a thực chất chỉ là mở rộng của đề Thanh Hóa TST 2016-2017 :)



#12
viet nam in my heart

viet nam in my heart

    Thượng sĩ

  • Điều hành viên OLYMPIC
  • 242 posts

Câu 5: b) Rõ ràng nếu gọi $A$ là tập các đoạn thẳng thỏa mãn đề bài và số phần tử của $A$ nhỏ nhất. Gọi $X$ là tập hợp tất cả các đoạn thẳng tạo từ $2$ trong số $42$ điểm trên. Khi đó $B=X \setminus A$ là một tập con của tập $X$ và nó không chứa bất cứ một tam giác nào

Do đó $|B| \leq \lfloor\dfrac{42^2}{4}\rfloor=441$ ( Theo định lý $Turan$)

Từ đó $|A| \geq \dfrac{42 \cdot 41}{2}-441=420$


"Nếu bạn hỏi một người giỏi trượt băng làm sao để thành công, anh ta sẽ nói với bạn: ngã, đứng dậy là thành công." Isaac Newton

VMF's Marathon Hình học Olympic


#13
vpvn

vpvn

    Binh nhất

  • Thành viên mới
  • 41 posts

câu hình 

a) dùng phương tích 

b)EP cắt QF tại U , PF cắt EQ taị V AN cắt BM tại T  $\Rightarrow$ UV là đường đối cực của T đối với (EFPQ).

(ADN) cắt (BDM) tại K $\Rightarrow$ T là trực tâm của KAB $\Rightarrow$ TD vuông góc AB .

từ đây được (ATEF)=(BTPQ)=-1 (1) $\Rightarrow$ A B thuộc đường đối cực của T đối với (EFPQ).

$\Rightarrow$ ABUV thẳng hàng

 (1) $\Rightarrow$ AB PE QF đồng qui tại U .

AQ cắt BF tại H áp dụng định lý Desargues cho 2 tam giác AQE BPF được H V T thẳng hàng 

Có (UVAB)=-1 đặt AP căt BE tại Z $\Rightarrow$ TZV thẳng hàng 

$\Rightarrow$ TZVH thẳng hàng  áp dụng định lý Desargues cho 2 tam giác AEY BPX  $\Rightarrow$ AB PE XY đồng qui

 $\Rightarrow$ đpcm



#14
takarin1512

takarin1512

    Trung sĩ

  • Thành viên
  • 104 posts

Bài 2:

a. Dùng quy nạp để chứng minh, câu này thì dễ rồi.

b. Xét hiệu $nx_n-\left ( n+1 \right )x_{n+1}=nx_n-\frac{n\left ( n+1 \right )x^2_n}{1+\left ( n+1 \right )x_n}=\frac{nx_n}{1+\left ( n+1 \right )x_n}$

$\Rightarrow y_n=x_1-\left ( n+1 \right )x_{n+1}$

Theo câu a thì $lim\left ( n+1 \right )x_{n+1}=0\Rightarrow limy_n=x_1=\frac{1}{2}$



#15
quangkhaiolympic

quangkhaiolympic

    Lính mới

  • Thành viên mới
  • 5 posts

câu này k là hằng số cố định nên phải đánh giá như sau thì mới hoàn tất

   do n chạy vô cùng nên đến lúc nào đó k<n . Vì vậy

           Yn>1/2 và Yn<vế đánh giá trên (xin lỗi mình ko bik gõ Latex) do đó theo NLK lim yn=1/2 :D  :D






1 user(s) are reading this topic

0 members, 1 guests, 0 anonymous users